please help me and before you try to answer the height is not 8 in but the volume is 25.12 in if that helps thank you <3

Please Help Me And Before You Try To Answer The Height Is Not 8 In But The Volume Is 25.12 In If That

Answers

Answer 1

Answer:

25.12 in^3.

Step-by-step explanation:

Volume of the cylinder = 3.14 * r^2 * h

= 3.14 * 2^2 * 8

The volume of the preserve

= 1/4 * 3.14 * 4 * 8

= 25.12 in^3.

Answer 2

Answer:

2 in

Step-by-step explanation:

so the original height is 8 in and you need to find the height if the jar was 1/4 full

half of 8 is 4

and half of 4 is 2

your final answer is 2 in

good luck :)

i hope this helps

have a nice day !!


Related Questions

What is the sum of the polynomials?
(6x + 7 + X ²) + (2x² - 3)
0 -X2+6x+4
3x2 +6x+4
9x + 4
O 9x2 +4

Answers

Answer:

B) 3x^2+6x+4

Step-by-step explanation:

(6x+7+x^2)+(2x^2-3)

6x+7+x^2+2x^2-3

3x^2+6x+7-3

3x^2+6x+4

The sum of the polynomials is 3x² + 6x + 4.

What are Polynomials?

Polynomials are mathematical expressions which consist of one or more terms involving variables and coefficients connected with operations like multiplication, subtraction, addition and natural number powers of variables.

Given are two polynomials whose sum has to be found.

(6x + 7 + x ²) and (2x² - 3).

The sum of the polynomials is,

(6x + 7 + x ²) + (2x² - 3)

We have to operate the like terms together.

= (6x) + (7 - 3) + (x² + 2x²)

= 6x + 4 + 3x²

= 3x² + 6x + 4

Hence the sum of the polynomials is 3x² + 6x + 4.

Learn more about Polynomials here :

https://brainly.com/question/20630027

#SPJ7

Hank and Rosa play the following game: A bag has 15 tiles in it, each with a letter from the phrase probability game on it. Hank and Rosa take turns drawing a tile, recording the letter, and placing the tile back in the bag. Hank earns a point if he draws a vowel. Rosa earns a point if she draws a consonant. They decide that the letter y can be a vowel or a consonant. Which statement best explains whether or not the game is fair?

A. The game is not fair because the probability of Hank drawing a winning letter is less than the probability of Rosa drawing a winning letter.

B. The game is fair because Hank and Rosa have the same probability of drawing a winning letter.

C. The game is fair because both Hank and Rosa will get a point if the letter y is drawn.

D. The game is not fair because the probability of Hank drawing a winning letter is greater than the probability of Rosa drawing a winning letter.

Answers

Answer:

A

Step-by-step explanation:

There are 8 consonants (not including y) and only 6 vowels (not including y), so Rosa has a higher probability of winning.

The game is not fair because the probability of Hank drawing a winning letter is less than the probability of Rosa drawing a winning letter.

What is probability?

'Probability is defined as the possibility of an event to happen is equal to the ratio of the number of outcomes to the total number of outcomes.'

According to the given problem,

There are 8 consonants (not including y) and only 6 vowels (not including y), so Rosa has a higher probability of winning.

Hence, we can conclude that the game is not fair the probability of Hank drawing a winning letter is less than the probability of Rosa drawing a winning letter.

Learn more about probability here:

https://brainly.com/question/11234923

#SPJ2

simplify the product using the distributive property.

(8b+5)(7b-2)=?

Answers

Answer:

5 6 2 + 1 9 − 1 0

Step-by-step explanation:

Angela tried to solve an equation step by step. \begin{aligned} \dfrac34+m&=\dfrac54\\\\ \dfrac34+m-\dfrac34&=\dfrac54+\dfrac34&\green{\text{Step } 1}\\\\ m&=2&\blue{\text{Step } 2} \end{aligned} 4 3 ​ +m 4 3 ​ +m− 4 3 ​ m ​ = 4 5 ​ = 4 5 ​ + 4 3 ​ =2 ​ Step 1 Step 2 ​

Answers

Answer:

Angela's mistake is in step 1, she failed to change the sign when subtracting from both sides

Step-by-step explanation:

Equation given :

The correct solution should be :

3/4 - 3/4 + m = 5/4 - 3/4

m = 2/ 4

m = 1/2

Therefore failure to change the sign will result in a obtaining a wrong answer for the exercise.

m=1/2

hope this helps!

Which of the following statements must be true based on the diagram below? Answer correctly !!!!!!!! Will mark brainliest !!!!!!!!!!!!

Answers

Answer:

3rd one

Step-by-step explanation:

Part B The manager of The Gadget Factory wants to make sure that the next shipment costs no more than $15,000. Write an inequality that shows the amount of money that can be spent. Graph the inequality on the number line. Let n=the amount of money spent.

Answers

Answer:

Part B The manager of The Gadget Factory wants to make sure that the next shipment costs no more than $15,000. Write an inequality that shows the amount of money that can be spent. Graph the inequality on the number line. Let n=the amount of money spent.

Step-by-step explanation:

The inequality equation is n ≤ 15,000 , where n is the amount of money spent

What is an Inequality Equation?

Inequalities are the mathematical expressions in which both sides are not equal. In inequality, unlike in equations, we compare two values. The equal sign in between is replaced by less than (or less than or equal to), greater than (or greater than or equal to), or not equal to sign.

In an inequality, the two expressions are not necessarily equal which is indicated by the symbols: >, <, ≤ or ≥.

Given data ,

Let the inequality equation be represented as A

Now , the value of A is

Let the amount of money spent be represented as n

And , the next shipment costs no more than $15,000

So , n ≤ 15,000   be equation (1)

Therefore , the value of A is n ≤ 15,000 and the graph is plotted

Hence , the inequality equation is n ≤ 15,000

To learn more about inequality equation click :

https://brainly.com/question/11897796

#SPJ2

Need help in math will give brain if right

Answers

Answer:

c)

Step-by-step explanation:

7.
Wendy is paid $12 per hour and plans to work between 30 and 35 hours per week. Identify the independent and dependent quantity in the situation. Find reasonable domain and range values.


A. weekly pay; hours worked; $360 to $420; 30 to 35 hours

B. hours worked; weekly pay; 30 to 35 hours; $360 to $420

C. weekly pay; hours worked; 30 to 35 hours; $360 to $420

D. hours worked; weekly pay; 30 to 35 hours; $0 to $420

Answers

Answer:   I'm gonna go with B. hours worked; weekly pay; 30 to 35 hours; $360 to $420, because I had something similar to this

Wendy is paid $9 per hour and plans to work between 20 and 25 hours per week. Identify the independent and dependent quantity in the situation. Find reasonable domain and range values.

 

weekly pay; hours worked; 20 to 25 hours; $180 to $225

 

hours worked; weekly pay; 20 to 25 hours; $180 to $225

 

weekly pay; hours worked; $180 to $225; 20 to 25 hours

 

hours worked; weekly pay; 20 to 25 hours; $9 to $225

Step-by-step explanation:

The independent quantity is her pay and the dependent quantity is the hours she works in a week.

I believe the range would be 30 to 35 hours and the domain would be $360 to $450.

...................................................................................................................................................

Answer:

Part a) The independent quantity is the variable x (the number of hours) and the dependent quantity is the variable y (total paid)

Part b) Domain  

Part c) Range  

Step-by-step explanation:

Let

x------> the number of hours

y-----> total paid

The linear equation that represent this situation is

so

Part a)

The independent quantity is the variable x (the number of hours)

The dependent quantity is the variable y (total paid)

Part b) Find the domain

The domain is the interval-------->  

All real numbers greater than or equal to  hours and less than or equal to  hours

Part c) Find the range

For  

find the value of y

For  

find the value of y

The range is the interval------->  

All real numbers greater than or equal to  and less than or equal to  

...............................................................................................................................................

The dependent quantity would be the $12 per hour, while the independent quantities are 30 hrs/ week to 35 hours/ week.

The $12 per hour would become $360/week to $420/week.

The domain of the problem, with the unit of hrs/week, would be :30,31,32,33,34,35.

The Range of the problem, with the unit of $/week, would be : 360, 372, 384, 396, 408, 420.

...............................................................................................................................................

If the center of a circle is at (a, b) and the radius of the circle is c, how would you write the equation of the circle?

Answers

Answer:

Step-by-step explanation:

(x-a)² + (y-b)² = c²

Which of the following sets of side lengths could produce a triangle? Select
all that apply.
13 mm, 5 mm, and 6 mm
2 cm, 7 cm, and 6 cm
2 m, 5 m, and 7 m
6 ft, 6 ft, and 12 ft

Answers

6ft, 6ft, and 12ft !

The sets of side lengths that can produce a triangle are

2 cm, 7 cm, and 6 cm2 m, 5 m, and 7 m6 ft, 6 ft, and 12 ft

The triangle inequality theorem states that the sum of any two sides of a triangle is greater than or equal to the third side.

That is, in a given triangle of sides a, b and c,

a + b ≥ c

Analysing the sets of the side lengths given,

In triangle 2 cm, 7 cm, and 6 cm,

2 + 6 = 8,

Therefore 8cm which is greater than the third side being 7cm makes this side lengths possible to produce a triangle.

In triangle 2 m, 5 m, and 7 m,

2 + 5 = 7

Addition of both sides of the triangle is equal to the third side being 7m.

In triangle 6 ft, 6 ft, and 12 ft,

6 + 6 = 12

Addition of both sides of the triangle is equal to the third side being 12ft

Therefore the sets of side lengths could produce a triangle are

2 cm, 7 cm, and 6 cm

2 m, 5 m, and 7 m

6 ft, 6 ft, and 12 ft

Learn more here:

https://brainly.com/question/18548908

what is 4/5 as a whole number

Answers

Answer: 4 over 5 is not a whole number and so it cannot be expressed as a whole number.

Step-by-step explanation:

Last year, there were 120 students in the
marching band. This year, there are 138.
What is the percentage of increase or decrease?

Answers

Answer:

The increase is 15%

Step-by-step explanation:

Hope this helps!

Explain what is a SYSTEM OF LINEAR EQUATIONS?

Answers

is just a set of two or more linear equations. In two variables (x and y) , the graph of a system of two equations is a pair of lines in the plane.

y=3(50)x
Based on the previous problem, how many bacteria will there be in 8 hours?
A. 74
B. 6,611
C. 1,200
D. 328, 050

Answers

Answer:

y=3(50)×

y=3(50) 8

50×3=150

150×8=1,200

(-15)-(-18) divided by 3

Answers

Answer:

The answer is (-9)

Step-by-step explanation:

The solution is, the result of the division is 1.

What is division?

Division is the process of splitting a number or an amount into equal parts. Division is one of the four basic operations of arithmetic, the ways that numbers are combined to make new numbers. The other operations are addition, subtraction, and multiplication.

here, we have,

given that,

(-15)-(-18)

now we have to divide this by 3

so, we get,

(-15)-(-18) /3

now,

(-15)-(-18)

= -15 + 18

=3

so, we have,

(-15)-(-18) /3

=3 /3

=1

Hence. The solution is, the result of the division is 1.

To learn more on division click:

brainly.com/question/21416852

#SPJ3

Please help me on my math assignment you have to write an inequality that models the combinations of multiple choice and essay questions you could answer during the test without running out of time.

Answers

Answer:

x + 15y ≤ t

Step-by-step explanation:

From the table we see that:

Each multiple choice question takes 1 min and each essay takes 15 min to complete.x multiple choice questions will take x min and y essays will take 15y minTotal time required is x + 15y

If you have t minutes to complete the test then required inequality is:

x + 15y ≤ t

What is the interest rate if…

Principal = $1,250

Time = 4 years

Interest = $285.

Answers

The interest rate is 5.7%

find the value of given expression

[tex] \sqrt{9 \times 81 \times 9} [/tex]

Answers

Answer:

√{9×81×9}=√(9²×9²)=±(9×9)=±81

Answer:

your answer will be 81

Step-by-step explanation:

.......

ixl i need help asap

Answers

Answer:

1. Baseball

2. Soccer ball

3. Football

Explantion:

Speed = Distance / time

Baseball --> 1270/32 = 39.69

Football --> 1199/44 = 27.25

Soccer ball --> 1270/44 = 28.86

Answer:

1. Baseball

2. Soccer ball

3. Football  

Formula= Speed = Distance / time

Baseball in fraction form is 1270/32 and in decimal form is 39.69

Football in fraction from is 1199/44 and in decimal form is 27.25

Soccer ball in fraction form is 1270/44 and in decimal 28.86

Hope this helps <3 Need thanks? Comment /hearthelp

What is the area of a circle with a radius of 4 cm? Give your answer in terms of pi.

(This is a math question from flocabulary, and I really need help!!)

Answers

Answer:

r = the radius, in your case, it's 4 cm. So. Area = π ×r2. = π× (4 cm)2. = 16π cm2 ≈ 50.24 cm2.

Step-by-step explanation:

a bed room set is being sold for $5500 if bought cash​

Answers

Answer:

$5700 - $5500= $200

Explanation:

The total installments: 24 ($175) = $4200

The hire purchase price: $1500 + $4200 = $5700

The amount saved by paying cash: $5700 - $5500= $200

Help me please thanks

Answers

Replace x any with the given values:

x/y = 9/3 = 3

The answer is 3

the sum of 2 consecutive integers is three times their difference what is the larger number​

Answers

Answer:

2

Step-by-step explanation:

Let the two consecutive integers be x , x+1

Sum : x + x +1 = 2x + 1

Difference: (x +1) - 1 = x + 1 - 1 = x

Sum = 3 times their difference

2x + 1 = 3 * x

2x + 1 = 3x    

      1 = 3x - 2x

     1 = x

The numbers are 1 , 2

I have a question on triangle congruence its a homework assignment ​

Answers

What’s the question man?

840 Km in 12 hours.Whatis the average speed.

Answers

ANSWER:
70 km per hour

STEP BY STEP:
divide 840/12

hope this helps

Winnie when she fell asleep sitting up so she just kept going back lol now I’m on a rampage posting my cats :,)

Answers

Answer:

that's an adorable little baby.

Step-by-step explanation:

Answer:

its cute

Step-by-step explanation:

are these triangles congruent ?

Answers

Answer:

yes

Step-by-step explanation:

from sss axiom

we can say that they are congurqnt

A mountain climber found that the relationship between his altitude and the surrounding air
temperature is linear. The function f (a) = - -100 a + 75 represents this relationship, where a
is the altitude in meters and f(a) is the temperature in degrees Fahrenheit. According to this
function, by how much is the temperature decreasing for every meter that the mountain climber's
altitude increases?
IF
100
O IF
100
75 F
-75F

Answers

Answer:- 1/100 degrees Fahrenheit

Step-by-step explanation:

The directions say decreasing so it has to be negative, and the slope is showing how much it is decreasing every meter. y=mx+b. The mx is the slope which is where the answer is located.

what is the slope of the line in the graph below ?

Answers

Answer:

Step-by-step explanation: The slope of a line is rise over run. Learn how to calculate the slope of the line in a graph  by  finding  the change  in y and the change in x.

Find the length of side x in the simplest radical form with a rational denominator.

Answers

Answer:

[tex]\sqrt{2}[/tex]

Step-by-step explanation:

We can use either angle, but I'm going to use the one on the bottom. So, in order to find x, we need to use tangent. One side we know is the adjacent, and the side we don't know is the opposite, therefore we need tangent. Here's the equation:

[tex]tan(45)=\frac{x}{\sqrt{2}}[/tex]

Obviously, we can't have a root in our denominator, so we need to get rid of it somehow. Here's how:

We multiply the denominator of the fraction by [tex]\sqrt{2}[/tex]. [tex]\sqrt{2}[/tex] multiplied by itself is simply 2. Try it! We also want to multiply the numerator by [tex]\sqrt{2}[/tex], but there isn't really a number we can use with that, so we'll just add it to the side. The equation you have now is:

[tex]tan(45)=\frac{x\sqrt{2} }{2}[/tex]

Let's try to work this out now. Since the denominator is 2, we have to multiply both sides by it to find x.

[tex]\frac{x\sqrt{2} }{2*2} =x\sqrt{2}[/tex]

[tex]tan(45)*2=2[/tex]

We can plug 2 in for the x in the numerator now:

[tex]\frac{2\sqrt{2} }{2}[/tex]

2 and 2 cancel out, so you get 1 in both the numerator and denominator. That's how we get our answer of [tex]x=\sqrt{2}[/tex]

Also, because this is a 45-45-90 triangle, you don't really have to do all that work. If it's a 45-45-90 triangle, both legs should be the same length. :)

Other Questions
pls help!due very soon.ill pick brainliest for the answer that helps me the most.(I can only pick brainliest if there's two answers) Read this sentence from lines 32 through 34 of the speech. "The events in Birmingham and elsewhere have so increased the cries for equality that no city or State or legislative body can prudently choose to ignorethem." How does this sentence help to develop a key concept in the speech?A) It informs the public about recent events in Birmingham.B) It explains why legislation has not yet been enacted.C) It shows how different it will be to make any kind of change to existing laws.D) It demonstrates the heightened need for action in preventing inequality Find the missing side. Round to the nearest tenth. How would you limit the domain tomake this function one to one?f(x) = (x + 3)^2 te quiero!!! Find the maximum rate of change of f at the given point and the direction in which it occurs. f(p, q) = 8qep + 4peq, (0, 0) maximum rate of change $$ Correct: Your answer is correct. direction $$ Which of these results in kinetic energy of an object? (1 point)OpositionO motionO massvolume analyse the impact that the increasing number of social grants may have on the unemployment rate The military reserves have contributed toward the nation's security and defense for several years. However, they have evolved in their role and function over the years. Research online and trace the history of the military reserves. Describe the difference in their role before and after the Vietnam War. what's the value of[tex] = > 25 \times 386 + 3219 \div 2[/tex] Is the relationship between the number of days and the number of hours proportional? why students should get back to the schools? One half of one one hundredth is PLEASE HELP ASAP PLEASE help please??????????????????????? Which of the following best describes the masters mood?angry because Hester disobeyed him and went out with Nedupset and hurt because none of his slaves like himconcerned for Hesters safety because she went out alone with a manconfused because none of his slaves have ever disobeyed him before Identify two employment laws which might affecteasyJet plc's business activities. CAN SOMEONE PLZZZZZZZ HELP ME ON THIS QUIZ IM BEGGING YALL ILL GIVE YALL BRAINLIST Given the function f (x) = x + 8x + 7, what are the zeros of f? One jar has 13 pennies, a second jar has 7 pennies, a 3rd jar has 10 pennies, and a 4th jar has 30 pennies. On average how many pennies does each jar have?